LSAT and Law School Admissions Forum

Get expert LSAT preparation and law school admissions advice from PowerScore Test Preparation.

User avatar
 KelseyWoods
PowerScore Staff
  • PowerScore Staff
  • Posts: 1079
  • Joined: Jun 26, 2013
|
#74405
Hi rk510!

As discussed above, answer choice (E) is supported by the linking the information in the stimulus together: the ozone filters out some wavelengths of lights, holes in the ozone layer are dangerous, one danger that has not been given enough attention is that these holes could lead to severe eye damage for animals of many species. So if the holes can lead to severe eye damage and the holes are basically just those areas where the ozone is not filtering out some wavelengths of light, that supports the statement that some wavelengths of lights that cause eye damage are ones that are filtered out when there are not holes in the ozone but that are not filtered out when there are holes in the ozone. We're making this inference by connecting the facts in the stimulus. Notice also that this stem is a Most Strongly Supported question, meaning that we can hold the correct answer to a slightly lower standard than we would if this was a Must Be True question.

Hope this helps!

Best,
Kelsey
 Kelly R
  • Posts: 35
  • Joined: May 08, 2020
|
#76046
Hi PS,

Sorry to re-hash this one, but I have a quick question regarding answer (E). Though I agree that this answer is most strongly supported by the stimulus, I was hesitant to select it as the stimulus suggests that the ozone holes *could* lead to severe eye damage, while E suggests that the sunlight definitively *causes* eye damage. This discrepancy in absoluteness of language gave me reason for pause, so wondering if someone can help clear this up. Thanks!
 Adam Tyson
PowerScore Staff
  • PowerScore Staff
  • Posts: 5164
  • Joined: Apr 14, 2011
|
#76478
Hey Kelly! The language in the last sentence of the stimulus is more certain than you may realize. It's saying that the holes absolutely have the ability to cause eye damage - "lead to" is the same as "causes." So what we have to ask ourselves is why the author is so sure that these holes create that potential? It can only be because they are letting in wavelengths that damage eyes, wavelengths that the ozone layer normally filters out. What else could make the holes in the ozone layer a threat to eyes?

The certainty in the answer is okay, because this is a Most Strongly Supported question, the softer variation of Must Be True. The answer doesn't have to be 100% proven by the stimulus, just supported by it, and none of the answers have any support while this answer has plenty.
User avatar
 SGD2021
  • Posts: 72
  • Joined: Nov 01, 2021
|
#91763
Hello, wouldn't we be able to eliminate choice D because of the phrase "a single wavelength of sunlight" which is too specific to be supported by the stimulus? Also, in the Logical Reasoning PowerScore bible explanation for this question (page 136 of 2020-2021 bible edition), it is said that we can eliminate answer choice D because it references "most" species when the stimulus "only discusses 'many' species." However, it is also said in the bible that, on the LSAT, both "most" and "many" can be interpreted to include "all." So, what is the difference between "most" and "many" on the LSAT?

Additionally, why is the word "could" ignored in the last sentence of the stimulus? Doesn't "could" suggest that the holes may not always lead to severe eye damage for animals?

Finally, is it always necessary to eliminate answer choices for the same reasons provided in the bibles or is it okay to eliminate choices based on our own reasoning?
 Adam Tyson
PowerScore Staff
  • PowerScore Staff
  • Posts: 5164
  • Joined: Apr 14, 2011
|
#91772
I'll answer your last question first, SGD2021: it's great if you find additional reasons for eliminating a wrong answer, or additional reasons for selecting a right answer, beyond what we might talk about in our books or courses! As long as your reasoning is valid, why not? We rarely take the position that our explanation is the ONLY explanation, just that we think it is the best one, or at least a sufficient one. So taking that back to your first question, you're absolutely right that "a single wavelength" is another good reason to reject answer D! In fact, it was the first one I noticed (so by the time I got to the word "most" in that answer I already knew it was a loser, and I barely paid any attention to that second problem). In other words, D is a terrible answer, filled with flaws. Nice work!

The difference between "most" and "many" is one of objective measurement. "Many" is completely subjective - it could be a tiny percentage of a very large number. For example, it might be reasonable to say that there are many LSAT tutors in Los Angeles, even those we make up only a tiny percentage of all people in L.A. and also a tiny percentage of all LSAT tutors in the world. "Most," on the other hand, has a clear, objective, measurable minimum requirement: "more than half." And while some cases of "most" and "many" overlap (many of my students are living, breathing human beings; most of them are; all of them are), some cases of "most" might not indicate "many" (most of my children are in the Navy, but since I have only one child, I don't have many in the Navy). The TL:DR version is that "many" is a subjectively large number and requires context to quantify, while "most" is guaranteed to be more than half of whatever number is under consideration and regardless of how big or small that number may be.

Finally, we aren't ignoring the "could" in the last sentence of the stimulus. If it's true that holes in the ozone layer could lead to eye damage, that means they have that possibility. It's certain that it's possible. And if the ozone layer is important because it filters out certain wavelengths of light, and if holes can lead to eye damage, then it is a reasonable inference (although not necessarily a guaranteed result) that the light let through the holes that would otherwise have been filtered out would be the cause of that damage. That's the only one of these answer choices that has ANY support, so it is the one that is MOST strongly supported. I think it's pretty darn near certain, given these facts, but even if you don't see it that way you still have to recognize that this answer is the best of the bunch and therefore must be the answer that we select. Pick the best answer, even if it seems imperfect to you!

Get the most out of your LSAT Prep Plus subscription.

Analyze and track your performance with our Testing and Analytics Package.